K
Khách

Hãy nhập câu hỏi của bạn vào đây, nếu là tài khoản VIP, bạn sẽ được ưu tiên trả lời.

19 tháng 9 2017

Áp dụng bất đẳng thức bu nhi a ta có 

\(\left(a^3+b^3+c^3+d^3\right)^2\le\left(a^4+b^4+c^4+d^4\right)\left(a^2+b^2+c^2+d^2\right)\)

=> \(\frac{a^4+b^4+c^4+d^4}{a^3+b^3+c^3+d^3}\ge\frac{a^3+b^3+c^3+d^3}{a^2+b^2+c^2+d^2}\)

tương tự ta có 

\(\frac{a^3+b^3+c^3+d^3}{a^2+b^2+c^2+d^2}\ge\frac{a^2+b^2+c^2+d^2}{a+b+c+d}\)

mà \(\left(a+b+c+d\right)^2\le\left(a^2+b^2+c^2+d^2\right)\left(1+1+1+1\right)\Rightarrow a^2+b^2+c^2+d^2\ge1\)

từ đó ta có 

\(\frac{a^4+b^4+c^4+d^4}{a^3+b^3+c^3+d^3}\ge\frac{1}{2}\)

dấu = xảy ra <=> \(a=b=c=d=\frac{1}{2}\)

19 tháng 9 2017

sai rồi bạn

10 tháng 7 2017

Áp dụng BĐT cauchy-schwarz :

\(VT=\frac{a^4}{ab+ac+ad}+\frac{b^4}{ab+bc+bd}+\frac{c^4}{cd+ac+bc}+\frac{d^4}{ad+bd+cd}\)

\(\ge\frac{\left(a^2+b^2+c^2+d^2\right)^2}{2\left(ab+ac+ad+bc+bd+cd\right)}\)

Mà \(3\left(a^2+b^2+c^2+d^2\right)\ge2\left(ab+ac+ad+bc+bd+cd\right)\)( dễ dàng chứng minh nó bằng AM-GM)

nên \(VT\ge\frac{a^2+b^2+c^2+d^2}{3}\)

Áp dụng BĐT AM-GM: \(a^2+b^2\ge2ab;b^2+c^2\ge2bc;c^2+d^2\ge2cd;d^2+a^2\ge2ad\)

\(\Rightarrow a^2+b^2+c^2+d^2\ge ab+bc+cd+da=1\)

do đó \(VT\ge\frac{1}{3}\)

Dấu''='' xảy ra khi \(a=b=c=d=\frac{1}{2}\)

7 tháng 9 2023

Trước tiên ta đi chứng minh BĐT phụ là:

Với �,�>0 thì �2+�4≥��(�2+�2)

Cách CM:

BĐT trên tương đương với: (�−�)2(�2+��+�2)≥0 (luôn đúng)

Quay trở về bài toán chính: Áp dụng BĐT phụ trên :

⇒��4+�4+�≤���(�2+�2)+�2��=���(�2+�2+�2)=�2�2+�2+�2

Thực hiện tương tự với các phân thức còn lại và cộng theo vế:

⇒�≤�2+�2+�2�2+�2+�2=1 (đpcm)

Dấu bằng xảy ra khi a=b=c=1

7 tháng 9 2023

loading...

Nó bị mất cái dấu gạch ngang chỗ phân số nha b

11 tháng 11 2018

giỏi thì làm bài nÀY nèk

chứ mấy bác cứ đăng linh ta linh tinh lên online math

11 tháng 11 2018

Linh ta linh tinh gì. ko biết làm thì tôi mới nhờ mọi người chứ

đây là câu cuối bài khảo sat trg tôi. ko làm được thì đừng phát biểu linh tinh

7 tháng 8 2015

Áp dụng Côsi:

\(a^4+a^4+a^4+1\ge4\sqrt[4]{\left(a^4\right)^3}=4a^3\)

\(\Rightarrow3\left(a^4+b^4+c^4+d^4\right)\ge4\left(a^3+b^3+c^3+d^3\right)-1\)

Ta chứng minh: \(a^3+b^3+c^3+d^3\ge4\)

Theo Côsi: \(a^3+1+1\ge3\sqrt[3]{a^3}=3a\)

\(\Rightarrow a^3+b^3+c^3+d^3+2.4\ge3\left(a+b+c+d\right)=3.4\)

\(\Rightarrow a^3+b^3+c^3+d^3\ge4\)

\(\Rightarrow3\left(a^4+b^4+c^4+d^4\right)\ge4\left(a^3+b^3+c^3+d^3\right)-4\ge3\left(a^3+b^3+c^3+d^3\right)\)

\(\Rightarrow a^4+b^4+c^4+d^4\ge a^3+b^3+c^3+d^3\)

31 tháng 3 2018

\(b^4+c^4\ge bc\left(b^2+c^2\right)\)vì \(\left(b-c\right)^2\left(b^2+bc+c^2\right)\ge0\)

\(\Rightarrow T\le\frac{a}{\frac{b^2+c^2}{a}+a}+\frac{b}{\frac{a^2+c^2}{b}+b}+\frac{c}{\frac{a^2+b^2}{c}+c}=1\)

1 tháng 4 2018

rõ đi bạn

9 tháng 12 2017

áp dụng bđt phụ

\(x^2+y^2+z^2>=xy+xz+yz\)

ta đượcp>=12

12 tháng 12 2017

nham. thuc ra

áp dụng bdt cô si ta có

\(\frac{a^4}{b\left(c+a\right)^2}+b>=\frac{a^2}{c+a}\)

cm tương tự 

do do P+a+b+c>=\(\frac{a^2}{c+a}+\frac{b^2}{a+b}+\frac{c^2}{b+c}\)

áp dụng bất đẳng thức bunhiacopxki ta có

\(\frac{a^2}{c+a}+\frac{b^2}{a+b}+\frac{c^2}{b+c}>=\frac{\left(a+b+c\right)^2}{2\left(a+b+c\right)}=\frac{a+b+c}{2}=\frac{12}{2}=6\)

=>P>=-6

dau = xay ra<=>

\(\hept{\begin{cases}\frac{a^4}{b\left(c+a\right)^2}=b\\\frac{b^4}{c\left(a+b\right)^2}=c\end{cases}}va\hept{\begin{cases}\frac{c^4}{a\left(b+c\right)^2}=c\\\frac{\left(c+a\right)^2}{a^2}=\frac{\left(a+b\right)^2}{b^2}=\frac{\left(b+c\right)^2}{c^2}\\a+b+c=12\end{cases}}\)

<=>a=b=c=4(tm)

1 tháng 8 2020

Xét \(\frac{a^3}{a^2+ab+b^2}-\frac{b^3}{a^2+ab+b^2}=\frac{\left(a-b\right)\left(a^2+ab+b^2\right)}{a^2+ab+b^2}=a-b\)

Tương tự, ta được: \(\frac{b^3}{b^2+bc+c^2}-\frac{c^3}{b^2+bc+c^2}=b-c\)\(\frac{c^3}{c^2+ca+a^2}-\frac{a^3}{c^2+ca+a^2}=c-a\)

Cộng theo vế của 3 đẳng thức trên, ta được: \(\left(\frac{a^3}{a^2+ab+b^2}+\frac{b^3}{b^2+bc+c^2}+\frac{c^3}{c^2+ca+a^2}\right)\)\(-\left(\frac{b^3}{a^2+ab+b^2}+\frac{c^3}{b^2+bc+c^2}+\frac{a^3}{c^2+ca+a^2}\right)=0\)

\(\Rightarrow\frac{a^3}{a^2+ab+b^2}+\frac{b^3}{b^2+bc+c^2}+\frac{c^3}{c^2+ca+a^2}\)\(=\frac{b^3}{a^2+ab+b^2}+\frac{c^3}{b^2+bc+c^2}+\frac{a^3}{c^2+ca+a^2}\)

Ta đi chứng minh BĐT phụ sau: \(a^2-ab+b^2\ge\frac{1}{3}\left(a^2+ab+b^2\right)\)(*)

Thật vậy: (*)\(\Leftrightarrow\frac{2}{3}\left(a-b\right)^2\ge0\)*đúng*

\(\Rightarrow2LHS=\Sigma_{cyc}\frac{a^3+b^3}{a^2+ab+b^2}=\Sigma_{cyc}\text{ }\frac{\left(a+b\right)\left(a^2-ab+b^2\right)}{a^2+ab+b^2}\)\(\ge\Sigma_{cyc}\text{ }\frac{\frac{1}{3}\left(a+b\right)\left(a^2+ab+b^2\right)}{a^2+ab+b^2}=\frac{1}{3}\text{​​}\Sigma_{cyc}\left[\left(a+b\right)\right]=\frac{2\left(a+b+c\right)}{3}\)

\(\Rightarrow LHS\ge\frac{a+b+c}{3}=RHS\)(Q.E.D)

Đẳng thức xảy ra khi a = b = c

P/S: Có thể dùng BĐT phụ ở câu 3a để chứng minhxD:

27 tháng 7 2020

1) ta chứng minh được \(\Sigma\frac{a^4}{\left(a+b\right)\left(a^2+b^2\right)}=\Sigma\frac{b^4}{\left(a+b\right)\left(a^2+b^2\right)}\)

\(VT=\frac{1}{2}\Sigma\frac{a^4+b^4}{\left(a+b\right)\left(a^2+b^2\right)}\ge\frac{1}{4}\Sigma\frac{a^2+b^2}{a+b}\ge\frac{1}{8}\Sigma\left(a+b\right)=\frac{a+b+c+d}{4}\)

bài 2 xem có ghi nhầm ko

NV
13 tháng 4 2021

BĐT này do giáo sư Vasile đề xuất, và đây là lời giải của ông ấy:

Do vai trò của các biến là như nhau, ko mất tính tổng quát, giả sử \(a^2=max\left\{a^2;b^2;c^2;d^2\right\}\)

\(\Rightarrow a^2\ge\dfrac{b^2+c^2+d^2}{3}\)

Đặt \(x^2=\dfrac{b^2+c^2+d^2}{3}\Rightarrow x^2\le a^2\) (1)

Đồng thời \(x^2=\dfrac{b^2+c^2+d^2}{3}\ge\dfrac{1}{9}\left(b+c+d\right)^2=\dfrac{a^2}{9}\Rightarrow a^2\le9x^2\) (2)

\(\left(1\right);\left(2\right)\Rightarrow\left(a^2-x^2\right)\left(a^2-9x^2\right)\le0\) (3)

Ta có:

\(b^4+c^4+d^4=\left(b^2+c^2+d^2\right)^2-2\left(b^2c^2+c^2d^2+b^2d^2\right)\le\left(b^2+c^2+d^2\right)^2-\dfrac{2}{3}\left(bc+cd+bd\right)^2\)

\(=\left(b^2+c^2+d^2\right)^2-\dfrac{1}{6}\left[\left(b+c+d\right)^2-\left(b^2+c^2+d^2\right)\right]^2=9x^4-\dfrac{1}{6}\left(a^2-3x^2\right)^2=\dfrac{45x^4+6a^2x^2-a^4}{6}\)

Do đó:

\(12\left(a^4+b^4+c^4+d^4\right)\le12a^4+12.\dfrac{45x^4+6a^2x^2-a^4}{6}=90x^4+12a^2x^2+10a^4\)

Nên ta chỉ cần chứng minh:

\(7\left(a^2+3x^2\right)^2\ge90x^4+12a^2x^2+10a^4\)

\(\Leftrightarrow a^4-10a^2x^2+9x^4\le0\)

\(\Leftrightarrow\left(a^2-9x^2\right)\left(a^2-x^2\right)\le0\) (đúng theo (3))

Vậy BĐT được chứng minh hoàn tất.

Dấu "=" xảy ra khi \(b=c=d=-\dfrac{a}{3}\) và các hoán vị của chúng